Sie sind auf Seite 1von 9

Version 340 Exam 3 302 sutclie (52200) 1

This print-out should have 27 questions.


Multiple-choice questions may continue on
the next column or page nd all choices
before answering.
001 10.0 points
You add a small amount of HCl to a solution
of 0.20 M HBrO and 0.20 M NaBrO. What do
you expect to happen?
1. The K
a
for HBrO will increase.
2. The H
+
ions will react with the HBrO
molecules.
3. The pH will increase slightly.
4. The [OH

] will decrease slightly. correct


Explanation:
002 10.0 points
For the polyprotic acid H
3
PO
4
(phospho-
ric acid), which of the following expressions
would correspond to K
a3
?
1. HPO
4
2
PO
4
3
+ H
+
correct
2. H
2
PO
4

HPO
4
2
+ H
+
3. H
3
PO
4
H
2
PO
4

+ H
+
4. PO
4
3
+ 3 H
+
H
3
PO
4
5. H
3
PO
4
PO
4
3
+ 3 H
+
Explanation:
For a triprotic acid, K
a3
will describe the
3rd (and nal) deprotonation event.
003 10.0 points
Consider the ionization constants
hypochlorous acid (HOCl) : K
a
= 3.5 10
8
;
ammonia (NH
3
) : K
b
= 1.810
5
. A solution
of ammonium hypochlorite (NH
4
OCl) is
1. acidic, because the anion in the salt hy-
drolyzes to a greater extent than the cation in
the salt.
2. basic, because the anion in the salt hy-
drolyzes to a greater extent than the cation in
the salt. correct
3. acidic, because the cation in the salt hy-
drolyzes to a greater extent than the anion in
the salt.
4. neutral, because the cation and the anion
in the salt hydrolyze to the same extent.
5. neutral, because NH
4
OCl is a weak
base/weak acid salt.
6. basic, because the cation in the salt hy-
drolyzes to a greater extent than the anion in
the salt.
Explanation:
Compare
NH
+
4
+ H
2
O

NH
3
+ H
3
O
+
K
a
=
1
K
b
=
1
1.8 10
5
= 55555.6
OCl

+ H
2
O

HOCl + OH

K
b
=
1
K
a
=
1
3.5 10
8
= 2.85714 10
7
The production of OH

occurs to the
greater extent, making the solution basic.
004 10.0 points
The K
a
of lactic acid is 1.4 x 10
4
. If you mix
100 mL of a 0.1 M solution of lactic acid with
4 mL of 1.0M solution of NaOH, what is the
pH of the resulting solution?
1. 10.15
2. 3.85
3. 3.67 correct
4. 4.02
5. 7.00
6. 9.98
Explanation:
Version 340 Exam 3 302 sutclie (52200) 2
A 100 mL solution of 0.1 M lactic acid will
contain 0.01 moles of lactic acid. 4mL of
a 1.0M solution of NaOH will contain .004
moles of OH

ions.
HA(aq) + OH

(aq) A

(aq) + H
2
O
pH = pK
a
+ log
[lactate]
[lacticacid]
pK
a
= -log(1.4 x 10
3
) = 3.85
pH = 3.85 + log
(.004)
(.006)
= 3.67
005 10.0 points
Consider the fractional composition dia-
gram for a generic diprotic acid H
2
A.
0.2
0.4
0.6
0.8
1.0
2 4 6 8 10 12 14
pH
F
r
a
c
t
i
o
n
a
l
C
o
m
p
o
s
i
t
i
o
n
What is the structure of the dominant
species at pH 5?
1. H
2
A
2. HA

correct
3. A
2
Explanation:
To the left of 3.3, the red graph representing
H
2
A is dominant.
Between 3.3 and 13.2, the blue graph rep-
resenting HA

is dominant.
To the right of 13.2, the green graph repre-
senting A
2
is dominant.
006 10.0 points
An indicator changes from red to blue going
from its acidic (nonionized) form to its basic
(ionized) form. The indicator has a K
a
of
3.2 10
5
. What color would be visible in a
solution with this indicator at pH 5.62?
1. red
2. blue correct
3. blueish-purple
4. reddish-purple
5. purple
Explanation:
The pK
a
is 4.5 so that is where you would
see purple (50/50 blend of red and blue). You
would see all red for pH 3.5 and lower, and
all blue for 5.5 and above. The 5.62 pH given
is denitely beyond the transition range and
you would see BLUE.
007 10.0 points
What is the pH of a solution which is
0.400 M in dimethylamine ((CH
3
)
2
NH) and
0.600 M in dimethylamine hydrochloride

(CH
3
)
2
NH
+
2
Cl

? K
b
for dimethylamine =
0.00074.
1. 11.05
2. 2.95
3. 3.31
4. 10.87
5. 11.21
6. 10.78
7. 10.69 correct
Explanation:
K
b
= 0.00074 K
w
= 1 10
14
[(CH
3
)
2
NH] = 0.4 M [(CH
3
)
2
NH
+
2
] = 0.6 M
K
a, (CH3)2NH
+
2
=
K
w
K
b, (CH3)2NH
Applying the Henderson-Hasselbalch equa-
tion
pH = pK
a
+ log

[(CH
3
)
2
NH]
[(CH
3
)
2
NH
+
2
]

= log

K
w
K
a

+ log

[(CH
3
)
2
NH]
[(CH
3
)
2
NH
+
2
]

Version 340 Exam 3 302 sutclie (52200) 3


= log

1 10
14
0.00074

+ log

0.4
0.6

= 10.6931
008 10.0 points
What is the pH of 3 10
9
M Ba(OH)
2
?
1. 5.78
2. 5.48
3. 8.22
4. 8.52
5. 7.03 correct
Explanation:
[Ba(OH)
2
] = 3 10
9
M K
w
= 1 10
14
Ba(OH)
2
completely dissociates:
Ba(OH)
2
(aq) Ba
2+
(aq) + 2 OH

(aq)
[OH

] = 2 [Ba(OH)
2
]
= 2 (3 10
9
) = 6 10
9
which is less than the [OH

] in pure water
(1 10
7
), so we must consider this concen-
tration:
H
2
O

H
+
+ OH

ini 1 10
7
1 10
7
6 10
9
+6 10
9
n 9.4 10
8
1.06 10
7
K
w
= [OH

] [H
+
]
[H
+
] =
K
w
[OH

]
=
1 10
14
1.06 10
7
= 9.43396 10
8
Thus
pH = log[H
+
]
= log(9.43396 10
8
) = 7.02531
Remember to check if your pH makes sense.
The usual error here is to just substitute the
[OH

] from the Ba(OH)


2
alone into the K
w
expression to nd [H
+
]; this gives an acidic
pH value for the base Ba(OH)
2
.
009 10.0 points
At the stoichiometric point in the titration of
0.260 M CH
3
NH
2
(aq) with 0.260 M HCl(aq),
1. [CH
3
NH
+
3
] = 0.260 M.
2. the pH is 7.0.
3. the pH is greater than 7.
4. the pH is less than 7. correct
5. [CH
3
NH
2
] = 0.130 M.
Explanation:
010 10.0 points
Below is the pH curve for a titration of a weak
base with 0.10 M HCl. What is the pK
b
of
the weak base?
1. 7.00
2. 5.3
3. 9.4
4. 4.6 correct
5. 8.7
Version 340 Exam 3 302 sutclie (52200) 4
6. 11.3
7. 2.7
Explanation:
The end point is at 60 mL which makes the
half way point 30 mL. The pH at this point is
at 9.4 which will be the pK
a
of the conjugate
acid of the weak base. The pK
b
will be 14
minus the pK
a
which results in 4.6.
011 10.0 points
Assume the molar solubility of silver chromate
(Ag
2
CrO
4
) is represented as x. Which of
the following expressions correctly expresses
the relationship between the molar solubility
of silver chromate and the solubility product
constant (K
sp
) for this compound?
1. K
sp
= x
2
2. K
sp
= 4 x
3
correct
3. K
sp
= 4 x
2
4. K
sp
= 2 x
3
5. K
sp
= 2 x
2
Explanation:
The equation for the dissociation of
Ag
2
CrO
4
is written as
Ag
2
CrO
4

2 Ag
+
+ CrO

4
.
The expression for K
sp
=

Ag
+

2

CrO

.
Since the molar solubility is x, K
sp
can be
rewritten (in terms of x) as
K
sp
= [2 x]
2
[x] = 4 x
3
.

Ag
+

= 2 x because for every salt molecule


that dissociates, 2 Ag
+
ions are produced.
012 10.0 points
What would be the nal pH if 0.0100 moles of
solid NaOH were added to 100 mL of a buer
solution containing 0.600 molar formic acid
(ionization constant = 1.8 10
4
) and 0.300
molar sodium formate?
1. 3.84
2. None of these
3. 3.44
4. 3.65 correct
5. 4.05
Explanation:
013 10.0 points
Which of the following indicators would be
most suitable for the titration of 0.10 M
(CH
3
)
3
N (K
b
= 6.510
5
) with 0.10 M HCl?
Transition ranges in pH units are given in
parentheses after the indicator name.
1. bromocresol green (3.8 - 5.4)
2. bromothymol blue (6.0 - 7.6)
3. phenolphthalein (8.0 - 9.6)
4. alizarin yellow (10.1 - 12.0)
5. chlorophenol red (4.8 - 6.4) correct
6. thymol blue (1.2 - 2.8)
Explanation:
At the equivalence point the solution will
be 0.05 M (CH
3
)
3
NH
+
which has a K
a
=
1.54 10
10
. This translates to a equivalence
point pH of 5.56. The chlorophenol red has
5.56 in the middle of its transition range.
014 10.0 points
If 100 mL of 0.2 M CHOOH and 400 mL of
0.5 M NaCHOO are mixed, what is the pH
of the resulting solution? Assume the pK
a
of
formic acid is 3.7.
1. 5.1
2. 2.2
3. 4.7 correct
Version 340 Exam 3 302 sutclie (52200) 5
4. 4.1
5. 3.7
Explanation:
pH = pK
a
+ log

[A

]/[HA]

= 3.7 + log (0.4/0.04)


= 3.7 + log (10)
= 3.7 + 1
= 4.7
015 10.0 points
Which of the following mixtures will be a
buer when dissolved in a liter of water?
1. 0.4 mol NH
3
and 0.4 mol HCl
2. 0.2 mol HBr and 0.1 mol NaOH
3. 0.3 mol NaCl and 0.3 mol HCl
4. 0.2 mol HF and 0.1 mol NaOH correct
5. 0.1 mol Ca(OH)
2
and 0.3 mol HI
Explanation:
Eliminate answers that are obviously incor-
rect. The choice with 0.2 mol HBr and 0.1
mol Ca(OH)
2
are strong acids and strong
bases respectively; therefore, NOT buers.
The choice with 0.3 mol NaCl is a combina-
tion of spectator ions and a strong acid; this
does not form a buer. Remaining for cal-
culation are choices with 0.4 mol NH
3
and
0.2 mol HF. Now perform the neutralizaton
calculations on the remaining possibilities:
Choice with 0.4 mol NH
3
NH
3
+ H
+

NH
4
+
Initial 0.4 0.4 0
Change 0.4 0.4 0.4
Final 0 0 0.4
Choice with 0.2 mol HF
HF + OH

+ H
2
O
Initial 0.2 0.1 0
Change 0.1 0.1 0.1
Final 0.1 0 0.1
The choice with 0.2 mol HF has both weak
acid and weak conjugate base left over, so it
is the buer solution.
016 10.0 points
Determine the pH of a 0.03 M solution of
NaH
2
PO
4
? Assume H
3
PO
4
has a pK
a1
of 2.1
and a pK
a2
of 7.2 and a pK
a3
of 12.7.
1. 7.40
2. 9.95
3. 4.36
4. 1.81
5. 7.11
6. 4.65 correct
Explanation:
For a solution composed of a single ampho-
teric species (H
2
PO
4

),
pH = 0.5(pK
a1
+pK
a2
) = 0.5(2.1+7.2) = 4.65
017 10.0 points
The equivalence point during titration of an
acid with a base may not occur at a pH of 7.0
because
1. the indicator may be one that changes
color at some other pH.
2. if a strong acid or base is used, it is com-
pletely ionized and therefore not neutral.
3. A solution containing a weak acid or a
weak base could never be neutral.
4. hydrolysis of the salt produced may make
the solution acidic or basic. correct
Version 340 Exam 3 302 sutclie (52200) 6
5. The statement is false; the pH must be
7.0 at the equivalence point.
Explanation:
If a weak acid or a weak base is involved
in the titration its conjugate which forms at
the equivalence point hydrolyzes to produce
excess H
3
O
+
in the case of a weak base or
excess OH

in the case of a weak acid.


weak acid:
HA + OH

H
2
O + A

+ H
2
O

HA + OH

or weak base:
B + H
3
O
+
H
2
O + BH
+
BH
+
+ H
2
O

B + H
3
O
+
018 10.0 points
A solution contains 1.00 mole of acetic acid
and has pH = 2.50. How many grams of
NaCH
3
COO3H
2
O (of MW 136) should be
dissolved in the solution to raise the pH to
4.00?
1. 35 g
2. 20 g
3. 10 g
4. 24 g correct
5. 15 g
Explanation:
Let cpd = NaCH
3
COO3H
2
O
acetic acid = 1 mol pH
ini
= 2.5
K
a
= 1.8 10
5
MW
cpd
= 136 g
pH
n
= 4
CH
3
COOH + H
2
O

CH
3
COO

+ H
3
O
+
[CH
3
COO

] = [H
3
O
+
]
= 10
2.5
= 0.00316228
K
a
=
[CH
3
COO

][H
3
O
+
]
[CH
3
COOH]
[CH
3
COOH] =
[CH
3
COO

][H
3
O
+
]
K
a
=
(0.00316228)
2
(1.8 10
5
)
= 0.555556 M CH
3
COOH
1.00 mol CH
3
COOH

1.00 L soln
0.555556 mol CH
3
COOH
= 1.8 L soln
pH
n
= pK
a
+ log

[cpd]
[CH
3
COOH]

= pK
a
+ log [cpd]
log [CH
3
COOH]
log [cpd] = pH
n
pK
a
+ log[CH
3
COOH]
= 4 [log(1.8 10
5
)]
+ log 0.555556
= 1 M
[cpd] = 0.1 mol
(1.8 L soln)
0.1 mol cpd
L soln

136 g cpd
1 mol cpd
= 24.48 g cpd
019 10.0 points
When ammonium chloride is added to
NH
3
(aq),
1. the K
b
increases.
2. the pH of the solution increases.
3. the pH of the solution decreases. correct
4. the equilibrium concentration of NH
3
(aq)
decreases.
5. the pH of the solution does not change.
Explanation:
020 10.0 points
K
sp
for CaF
2
is 3.9 10
11
. Would a pre-
cipitate of CaF
2
form if Ca(NO
3
)
2
and NaF
Version 340 Exam 3 302 sutclie (52200) 7
solutions were mixed such that [Ca
2+
] =
2.0 10
4
M, and [F

] = 3.0 10
4
M?
1. yes, because Q is smaller than K
sp
2. no correct
3. yes, because Q is larger than K
sp
Explanation:
021 10.0 points
Calculate the pH of the solution resulting
from the addition of 29.0 mL of 0.180 M
HClO
4
to 73.0 mL of 0.140 M NaOH.
1. 12.84
2. 1.16
3. 13.15
4. 1.31
5. 12.69 correct
6. 7.00
7. 0.85
Explanation:
V
HClO4
= 29.0 mL [HClO
4
] = 0.180 M
V
NaOH
= 73.0 mL [NaOH] = 0.140 M
Here its important to nd out which of
these two species (HClO
4
and NaOH) is in ex-
cess. The one that is in excess will determine
the pH of this solution. From the formulas of
the two compounds, you can expect that they
will react in a one-to-one fashion.
So our rst order of business will be to
determine how many moles of each compound
we have.
For HClO
4
, we have
(29.0 mL)

1 L
1000 mL

0.180 mol
1 L

= 0.00522 mol HClO


4
Likewise, for NaOH, we have
(73.0 mL)

1 L
1000 mL

0.140 mol
1 L

= 0.0102 mol NaOH


So when HClO
4
and NaOH react, all of
the HClO
4
will be consumed (its the limiting
reagent) and
0.0102 mol 0.00522 mol = 0.00500 mol
of NaOH will remain. This 0.00500 mol ex-
cess of NaOH will determine the pH of this
solution. The solution now is
29.0 mL + 73.0 mL = 102 mL
and, since NaOH is a strong base (i.e., it is
completely dissociated), it contains 0.00500
mol OH

. [OH

] is then
[OH

] =
0.00500 mol
0.102 L
= 0.0490 M,
which means that the pOH of this solution is
pOH = log [OH

] = log 0.0490 = 1.31


However, we wanted pH. We can use the equa-
tion that relates pH to pOH to get the pH:
pH = 14 pOH = 14 1.31 = 12.69
022 10.0 points
Calculate the resulting pHobtained by mixing
16.5 g HCl and 22.6 g NaCl in water to create
482 L solution.
1. 3.92
2. 4.31
3. 5.67
4. 2.15
5. 1.13
6. 3.03 correct
Explanation:
m
HCl
= 16.5 g m
NaCl
= 22.6 g
V = 482 L
The resulting pH is obtained from the [H
+
].
? mol H
+
= 16.5 g HCl
1 mol HCl
36.45 g HCl

1 mol H
+
1 mol HCl

1
482 L
= 9.39 10
4
M H
+
Version 340 Exam 3 302 sutclie (52200) 8
pH = log[H
+
]
= log[9.39 10
4
] = 3.03
023 10.0 points
Explain why the salt of a weak acid, as well
as the acid itself, must be present to form a
buer solution.
1. The cation from the salt is needed to
partially neutralize added acid.
2. The anion from the salt is needed to
partially neutralize added base.
3. The cation from the salt is needed to
partially neutralize added base.
4. Actually, a weak acid by itself is a buer;
no salt is needed.
5. The anion from the salt is needed to
partially neutralize added acid. correct
Explanation:
The salt of the acid provides the anion
which is the conjugate base of the buer sys-
tem:
HA + H
2
O

A

+ H
3
O
+
This anion A

reacts with any added acid


(H
3
O
+
) to prevent any appreciable change in
pH.
024 10.0 points
I found some potassium chlorate
from inside our lab.
I measured some out on a big marble slab.
I dissolved it in water,
and took a breath of fresh air.
The pH
No big surprise there.
1. remained neutral. correct
2. was acidic.
3. was now basic.
Explanation:
Potassium chlorate is the salt of a strong
acid and strong base and is therefore a NEU-
TRAL salt.
025 10.0 points
What is the pH of a 0.34 M solution of anilin-
ium nitrate (C
6
H
5
NH
3
NO
3
)? K
b
for aniline
(C
6
H
5
NH
2
) is 4.2 10
10
.
1. 2.40
2. 4.92
3. 2.68
4. 2.31
5. 2.55 correct
6. 9.08
Explanation:
M
C6H5NH3NO3
= 0.34 M K
b
= 4.2 10
10
C
6
H
5
NH
3
NO
3
a salt of a weak base (BHX).
This means you need a K
a
for the weak acid
BH
+
:
K
a
=
K
w
K
b
=
1.0 10
14
4.2 10
10
= 2.38095 10
5
You CAN use the approximation for the
equilibrium which means that
[H
+
] =

K
a
C
BH
+
=

(2.38095 10
5
) (0.34)
= 0.00284521 M
pH = log(0.00284521) = 2.54589
This means you need a K
a
for the weak
acid BH
+
. Use K
a
=
K
w
K
b
to get the K
a
=
2.38095 10
5
. You can use the approxima-
tion for the equilibrium which means that
[H
+
] =

K
a
C
BH
+ = 0.00284521 M
pH = log(0.00284521) = 2.54589
026 10.0 points
Consider a solution that is 0.10 M in a weak
Version 340 Exam 3 302 sutclie (52200) 9
triprotic acid which is represented by the gen-
eral formula H
3
A with ionization constants
K
1
= 1.0 10
3
, K
2
= 1.0 10
8
, and
K
3
= 1.0 10
12
. What is the pH of the
solution?
1. 2.25
2. 2.02 correct
3. 2.35
4. 2.15
5. 2.54
Explanation:
027 10.0 points
Consider the titration of 15.0 mL of 0.200 M
H
3
PO
4
(aq) with 0.200 M NaOH(aq). What
is/are the major species in solution after the
addition of 15.0 mL of base?
1. H
2
PO

4
(aq) correct
2. H
3
PO
4
(aq) and H
2
PO

4
(aq)
3. PO
3
4
(aq)
4. H
2
PO

4
(aq) and HPO
2
4
(aq)
Explanation:

Das könnte Ihnen auch gefallen